38fdf693ae72edd6c936043f90164cb259365415
[course.git] / latex / problems / Giancoli_6 / problem19.24.tex
1 \begin{problem*}{24}
2 Determine the terminal voltage of each battery in Fig.~19-44.
3 \begin{center}
4 \begin{asy}
5   import Circ;
6   real u = 3cm;
7   TwoTerminal ra = resistor((0,0), normal, 0, "$r_1 = 1.0\U{\Ohm}$", "");
8   TwoTerminal Ba = source(ra.end, DC, 0, "", "$\mathcal{E}_1 = 12\U{V}$");
9   TwoTerminal rb = resistor(ra.beg+(0,-u), normal, 0, "$r_2 = 2.0\U{\Ohm}$", "");
10   TwoTerminal Bb = source(rb.end, DC, 0, "", "$\mathcal{E}_2 = 18\U{V}$");
11   TwoTerminal R = resistor(Bb.end+(0.5u,0.25u), normal, 90, "", "$R = 6.6\U{\Ohm}$");
12   wireU(rb.beg, ra.beg, -24pt, rlsq);
13   wire(Bb.end, R.beg, rlsq);
14   wire(Ba.end, R.end, rlsq);
15 \end{asy}
16 \end{center}
17 \end{problem*}
18
19 \begin{solution}
20 From Kirchoff's loop rule
21 \begin{align*}
22   \mathcal{E}_1 - IR - \mathcal{E}_2 - Ir_2 - Ir_1 &= 0 \\
23   I(R+r_1+R_2) &= \mathcal{E}_1-\mathcal{E}_2 \\
24   I &= \frac{\mathcal{E}_1-\mathcal{E}_2}{R+r_1+r_2} = 0.625\U{A}
25 \end{align*}
26
27 So the voltage across the top battery is
28 $$V_1 = \mathcal{E}_1 - I r_1 = \ans{17\U{V}} \qquad (17.375\U{V})$$
29 and the voltage across the bottom battery is
30 $$V_2 = \mathcal{E}_2 - I r_2 = \ans{11\U{V}} \qquad (10.75\U{V})$$
31 \end{solution}